Partial sums of the series $sumlimits_kgeq1frac1sqrt2k+sqrt4k^2-1$

The name of the pictureThe name of the pictureThe name of the pictureClash Royale CLAN TAG#URR8PPP











up vote
3
down vote

favorite
2












The series $sumlimits_k=1^inftyfrac1sqrt2k+sqrt4k^2-1$ is divergent. I am interested in its partial sums to do some computations based on them.



I tried to multiply $sqrt2k+sqrt4k^2-1$ by $sqrt2k-sqrt4k^2-1$ and to divide by the same term, but I have got no telescoping sum for evaluation. Maybe I should use others technics.



Now my question is what is the closed form of the sum $$sum_k=1^nfrac1sqrt2k+sqrt4k^2-1 ?$$







share|cite|improve this question

























    up vote
    3
    down vote

    favorite
    2












    The series $sumlimits_k=1^inftyfrac1sqrt2k+sqrt4k^2-1$ is divergent. I am interested in its partial sums to do some computations based on them.



    I tried to multiply $sqrt2k+sqrt4k^2-1$ by $sqrt2k-sqrt4k^2-1$ and to divide by the same term, but I have got no telescoping sum for evaluation. Maybe I should use others technics.



    Now my question is what is the closed form of the sum $$sum_k=1^nfrac1sqrt2k+sqrt4k^2-1 ?$$







    share|cite|improve this question























      up vote
      3
      down vote

      favorite
      2









      up vote
      3
      down vote

      favorite
      2






      2





      The series $sumlimits_k=1^inftyfrac1sqrt2k+sqrt4k^2-1$ is divergent. I am interested in its partial sums to do some computations based on them.



      I tried to multiply $sqrt2k+sqrt4k^2-1$ by $sqrt2k-sqrt4k^2-1$ and to divide by the same term, but I have got no telescoping sum for evaluation. Maybe I should use others technics.



      Now my question is what is the closed form of the sum $$sum_k=1^nfrac1sqrt2k+sqrt4k^2-1 ?$$







      share|cite|improve this question













      The series $sumlimits_k=1^inftyfrac1sqrt2k+sqrt4k^2-1$ is divergent. I am interested in its partial sums to do some computations based on them.



      I tried to multiply $sqrt2k+sqrt4k^2-1$ by $sqrt2k-sqrt4k^2-1$ and to divide by the same term, but I have got no telescoping sum for evaluation. Maybe I should use others technics.



      Now my question is what is the closed form of the sum $$sum_k=1^nfrac1sqrt2k+sqrt4k^2-1 ?$$









      share|cite|improve this question












      share|cite|improve this question




      share|cite|improve this question








      edited Jul 22 at 20:50









      Did

      242k23208443




      242k23208443









      asked Jul 22 at 15:11









      zeraoulia rafik

      2,1071823




      2,1071823




















          1 Answer
          1






          active

          oldest

          votes

















          up vote
          9
          down vote



          accepted










          Using the identity



          $$sqrta+sqrtb=sqrtfrac12 left(a+sqrta^2-bright)+sqrtfrac12 left(a-sqrta^2-bright)$$



          where: $a = 2 k$ and $b=4 k^2-1$, along with evaluating a telescoping series, we find that



          $$beginalign
          colorredsum _k=1^n frac1sqrt2 k+sqrt4 k^2-1&=sum _k=1^n fracsqrt2sqrt-1+2 k+sqrt1+2 k\\
          &=sum _k=1^n fracsqrt2
          left(sqrt2 k+1-sqrt2 k-1right)left(sqrt-1+2 k+sqrt1+2 kright) left(sqrt2 k+1-sqrt2 k-1right)\\
          &=fracsum _k=1^n
          left(sqrt2 k+1-sqrt2 k-1right)sqrt2\\
          &=frac-1+sqrt1+2 nsqrt2\\
          &=colorred-frac1sqrt2+fracsqrt1+2 nsqrt2
          endalign$$






          share|cite|improve this answer























          • Thanks so much for the slick answer
            – zeraoulia rafik
            Jul 22 at 17:09










          • @zeraouliarafik . You're welcome :)
            – Mariusz Iwaniuk
            Jul 22 at 17:20






          • 1




            (+1) Nicely done. But CAS is unnecessary. Simply multiply numerator and denominator by $sqrt(2k+1)-sqrt(2k-1)$ to yield a telescoping series.
            – Mark Viola
            Jul 22 at 18:18










          • @MarkViola. Thanks for HINT.
            – Mariusz Iwaniuk
            Jul 22 at 18:40










          • You're welcome. My pleasure.
            – Mark Viola
            Jul 23 at 21:43










          Your Answer




          StackExchange.ifUsing("editor", function ()
          return StackExchange.using("mathjaxEditing", function ()
          StackExchange.MarkdownEditor.creationCallbacks.add(function (editor, postfix)
          StackExchange.mathjaxEditing.prepareWmdForMathJax(editor, postfix, [["$", "$"], ["\\(","\\)"]]);
          );
          );
          , "mathjax-editing");

          StackExchange.ready(function()
          var channelOptions =
          tags: "".split(" "),
          id: "69"
          ;
          initTagRenderer("".split(" "), "".split(" "), channelOptions);

          StackExchange.using("externalEditor", function()
          // Have to fire editor after snippets, if snippets enabled
          if (StackExchange.settings.snippets.snippetsEnabled)
          StackExchange.using("snippets", function()
          createEditor();
          );

          else
          createEditor();

          );

          function createEditor()
          StackExchange.prepareEditor(
          heartbeatType: 'answer',
          convertImagesToLinks: true,
          noModals: false,
          showLowRepImageUploadWarning: true,
          reputationToPostImages: 10,
          bindNavPrevention: true,
          postfix: "",
          noCode: true, onDemand: true,
          discardSelector: ".discard-answer"
          ,immediatelyShowMarkdownHelp:true
          );



          );








           

          draft saved


          draft discarded


















          StackExchange.ready(
          function ()
          StackExchange.openid.initPostLogin('.new-post-login', 'https%3a%2f%2fmath.stackexchange.com%2fquestions%2f2859489%2fpartial-sums-of-the-series-sum-limits-k-geq1-frac1-sqrt2k-sqrt4k2-1%23new-answer', 'question_page');

          );

          Post as a guest






























          1 Answer
          1






          active

          oldest

          votes








          1 Answer
          1






          active

          oldest

          votes









          active

          oldest

          votes






          active

          oldest

          votes








          up vote
          9
          down vote



          accepted










          Using the identity



          $$sqrta+sqrtb=sqrtfrac12 left(a+sqrta^2-bright)+sqrtfrac12 left(a-sqrta^2-bright)$$



          where: $a = 2 k$ and $b=4 k^2-1$, along with evaluating a telescoping series, we find that



          $$beginalign
          colorredsum _k=1^n frac1sqrt2 k+sqrt4 k^2-1&=sum _k=1^n fracsqrt2sqrt-1+2 k+sqrt1+2 k\\
          &=sum _k=1^n fracsqrt2
          left(sqrt2 k+1-sqrt2 k-1right)left(sqrt-1+2 k+sqrt1+2 kright) left(sqrt2 k+1-sqrt2 k-1right)\\
          &=fracsum _k=1^n
          left(sqrt2 k+1-sqrt2 k-1right)sqrt2\\
          &=frac-1+sqrt1+2 nsqrt2\\
          &=colorred-frac1sqrt2+fracsqrt1+2 nsqrt2
          endalign$$






          share|cite|improve this answer























          • Thanks so much for the slick answer
            – zeraoulia rafik
            Jul 22 at 17:09










          • @zeraouliarafik . You're welcome :)
            – Mariusz Iwaniuk
            Jul 22 at 17:20






          • 1




            (+1) Nicely done. But CAS is unnecessary. Simply multiply numerator and denominator by $sqrt(2k+1)-sqrt(2k-1)$ to yield a telescoping series.
            – Mark Viola
            Jul 22 at 18:18










          • @MarkViola. Thanks for HINT.
            – Mariusz Iwaniuk
            Jul 22 at 18:40










          • You're welcome. My pleasure.
            – Mark Viola
            Jul 23 at 21:43














          up vote
          9
          down vote



          accepted










          Using the identity



          $$sqrta+sqrtb=sqrtfrac12 left(a+sqrta^2-bright)+sqrtfrac12 left(a-sqrta^2-bright)$$



          where: $a = 2 k$ and $b=4 k^2-1$, along with evaluating a telescoping series, we find that



          $$beginalign
          colorredsum _k=1^n frac1sqrt2 k+sqrt4 k^2-1&=sum _k=1^n fracsqrt2sqrt-1+2 k+sqrt1+2 k\\
          &=sum _k=1^n fracsqrt2
          left(sqrt2 k+1-sqrt2 k-1right)left(sqrt-1+2 k+sqrt1+2 kright) left(sqrt2 k+1-sqrt2 k-1right)\\
          &=fracsum _k=1^n
          left(sqrt2 k+1-sqrt2 k-1right)sqrt2\\
          &=frac-1+sqrt1+2 nsqrt2\\
          &=colorred-frac1sqrt2+fracsqrt1+2 nsqrt2
          endalign$$






          share|cite|improve this answer























          • Thanks so much for the slick answer
            – zeraoulia rafik
            Jul 22 at 17:09










          • @zeraouliarafik . You're welcome :)
            – Mariusz Iwaniuk
            Jul 22 at 17:20






          • 1




            (+1) Nicely done. But CAS is unnecessary. Simply multiply numerator and denominator by $sqrt(2k+1)-sqrt(2k-1)$ to yield a telescoping series.
            – Mark Viola
            Jul 22 at 18:18










          • @MarkViola. Thanks for HINT.
            – Mariusz Iwaniuk
            Jul 22 at 18:40










          • You're welcome. My pleasure.
            – Mark Viola
            Jul 23 at 21:43












          up vote
          9
          down vote



          accepted







          up vote
          9
          down vote



          accepted






          Using the identity



          $$sqrta+sqrtb=sqrtfrac12 left(a+sqrta^2-bright)+sqrtfrac12 left(a-sqrta^2-bright)$$



          where: $a = 2 k$ and $b=4 k^2-1$, along with evaluating a telescoping series, we find that



          $$beginalign
          colorredsum _k=1^n frac1sqrt2 k+sqrt4 k^2-1&=sum _k=1^n fracsqrt2sqrt-1+2 k+sqrt1+2 k\\
          &=sum _k=1^n fracsqrt2
          left(sqrt2 k+1-sqrt2 k-1right)left(sqrt-1+2 k+sqrt1+2 kright) left(sqrt2 k+1-sqrt2 k-1right)\\
          &=fracsum _k=1^n
          left(sqrt2 k+1-sqrt2 k-1right)sqrt2\\
          &=frac-1+sqrt1+2 nsqrt2\\
          &=colorred-frac1sqrt2+fracsqrt1+2 nsqrt2
          endalign$$






          share|cite|improve this answer















          Using the identity



          $$sqrta+sqrtb=sqrtfrac12 left(a+sqrta^2-bright)+sqrtfrac12 left(a-sqrta^2-bright)$$



          where: $a = 2 k$ and $b=4 k^2-1$, along with evaluating a telescoping series, we find that



          $$beginalign
          colorredsum _k=1^n frac1sqrt2 k+sqrt4 k^2-1&=sum _k=1^n fracsqrt2sqrt-1+2 k+sqrt1+2 k\\
          &=sum _k=1^n fracsqrt2
          left(sqrt2 k+1-sqrt2 k-1right)left(sqrt-1+2 k+sqrt1+2 kright) left(sqrt2 k+1-sqrt2 k-1right)\\
          &=fracsum _k=1^n
          left(sqrt2 k+1-sqrt2 k-1right)sqrt2\\
          &=frac-1+sqrt1+2 nsqrt2\\
          &=colorred-frac1sqrt2+fracsqrt1+2 nsqrt2
          endalign$$







          share|cite|improve this answer















          share|cite|improve this answer



          share|cite|improve this answer








          edited Jul 23 at 21:44









          Mark Viola

          126k1172167




          126k1172167











          answered Jul 22 at 15:56









          Mariusz Iwaniuk

          1,6511615




          1,6511615











          • Thanks so much for the slick answer
            – zeraoulia rafik
            Jul 22 at 17:09










          • @zeraouliarafik . You're welcome :)
            – Mariusz Iwaniuk
            Jul 22 at 17:20






          • 1




            (+1) Nicely done. But CAS is unnecessary. Simply multiply numerator and denominator by $sqrt(2k+1)-sqrt(2k-1)$ to yield a telescoping series.
            – Mark Viola
            Jul 22 at 18:18










          • @MarkViola. Thanks for HINT.
            – Mariusz Iwaniuk
            Jul 22 at 18:40










          • You're welcome. My pleasure.
            – Mark Viola
            Jul 23 at 21:43
















          • Thanks so much for the slick answer
            – zeraoulia rafik
            Jul 22 at 17:09










          • @zeraouliarafik . You're welcome :)
            – Mariusz Iwaniuk
            Jul 22 at 17:20






          • 1




            (+1) Nicely done. But CAS is unnecessary. Simply multiply numerator and denominator by $sqrt(2k+1)-sqrt(2k-1)$ to yield a telescoping series.
            – Mark Viola
            Jul 22 at 18:18










          • @MarkViola. Thanks for HINT.
            – Mariusz Iwaniuk
            Jul 22 at 18:40










          • You're welcome. My pleasure.
            – Mark Viola
            Jul 23 at 21:43















          Thanks so much for the slick answer
          – zeraoulia rafik
          Jul 22 at 17:09




          Thanks so much for the slick answer
          – zeraoulia rafik
          Jul 22 at 17:09












          @zeraouliarafik . You're welcome :)
          – Mariusz Iwaniuk
          Jul 22 at 17:20




          @zeraouliarafik . You're welcome :)
          – Mariusz Iwaniuk
          Jul 22 at 17:20




          1




          1




          (+1) Nicely done. But CAS is unnecessary. Simply multiply numerator and denominator by $sqrt(2k+1)-sqrt(2k-1)$ to yield a telescoping series.
          – Mark Viola
          Jul 22 at 18:18




          (+1) Nicely done. But CAS is unnecessary. Simply multiply numerator and denominator by $sqrt(2k+1)-sqrt(2k-1)$ to yield a telescoping series.
          – Mark Viola
          Jul 22 at 18:18












          @MarkViola. Thanks for HINT.
          – Mariusz Iwaniuk
          Jul 22 at 18:40




          @MarkViola. Thanks for HINT.
          – Mariusz Iwaniuk
          Jul 22 at 18:40












          You're welcome. My pleasure.
          – Mark Viola
          Jul 23 at 21:43




          You're welcome. My pleasure.
          – Mark Viola
          Jul 23 at 21:43












           

          draft saved


          draft discarded


























           


          draft saved


          draft discarded














          StackExchange.ready(
          function ()
          StackExchange.openid.initPostLogin('.new-post-login', 'https%3a%2f%2fmath.stackexchange.com%2fquestions%2f2859489%2fpartial-sums-of-the-series-sum-limits-k-geq1-frac1-sqrt2k-sqrt4k2-1%23new-answer', 'question_page');

          );

          Post as a guest













































































          Comments

          Popular posts from this blog

          Color the edges and diagonals of a regular polygon

          Relationship between determinant of matrix and determinant of adjoint?

          What is the equation of a 3D cone with generalised tilt?